Question

1. (25 points) Consider two firms, 1 and 2, producing an identical good simul taneously. This good has market demand given by

0 0
Add a comment Improve this question Transcribed image text
Answer #1

Ә) p32-37 п, - ру, – 29. — 1 п, 2 (12 - 3 y +4 ) 9- 29 - 1 п, : 12 - бу — 39 - о 10 — 69 - 3 - 0 10 -39 6 firm Is reaction f

phplhzmiu.png

(IIIIIIII).

Equilibrium in Substitute Yz in Yi 9. - 10 – 3 | 10 - 371) 69 = 10 - 10 -29. ) = 20 – 10+ 39 12 )) 10 +3y. - 49, - ю а Ну, у

for collusine equilibri MR=MC Revenue: 19 12-34) 4 MR: d Revenue pau - MR = 12-64 Marginal cost :dc:d (24,+1) = 2.1 - dydy, -

Profit in collusion is greater than non- collusion. Let firm I increase its output from 5 TD the price changes P: 19-35+10) 1

Add a comment
Know the answer?
Add Answer to:
1. (25 points) Consider two firms, 1 and 2, producing an identical good simul taneously. This...
Your Answer:

Post as a guest

Your Name:

What's your source?

Earn Coins

Coins can be redeemed for fabulous gifts.

Not the answer you're looking for? Ask your own homework help question. Our experts will answer your question WITHIN MINUTES for Free.
Similar Homework Help Questions
  • Consider the Cournot duopoly model in which two firms, 1 and 2, simul- taneously choose the...

    Consider the Cournot duopoly model in which two firms, 1 and 2, simul- taneously choose the quantities they supply, q1 and q2. The price each will face is determined by the market demand function (q1, q2) = a − b(q1 + q2). Each firm has a probability μ of having a marginal unit cost of cL, and a probability 1 − μ of having a marginal unit cost of cH, cH > cL. These probabilities are common knowledge, but the...

  • 5. Consider a version of the Cournot duopoly game, where firms 1 and 2 simul taneously...

    5. Consider a version of the Cournot duopoly game, where firms 1 and 2 simul taneously and independently select quantities to produce in a market. The quantity selected by firm i is denoted q, and must be greater than or equal to zero, for i -1,2. The market price is given by p - 100 - 2q Suppose that each firm produces at a cost of 20 per unit. Further, assume that each firm's payoff is defined as its profit....

  • Suppose there are two firms in a market producing differentiated products. Both firms have MC=0. The...

    Suppose there are two firms in a market producing differentiated products. Both firms have MC=0. The demand for firm 1 and 2’s products are given by: q1(p1,p2) = 5 - 2p1 + p2 q2(p1,p2) = 5 - 2p2 + p1 a. First, suppose that the two firms compete in prices (i.e. Bertrand). Compute and graph each firm’s best response functions. What is the sign of the slope of the firms’ best-response functions? Are prices strategic substitutes or complements? b. Solve...

  • 2. (30 pts) There are two firms in a market, producing the same good. The firms...

    2. (30 pts) There are two firms in a market, producing the same good. The firms simultaneously choose their output levels, qı for firm 1 and q2 for firm 2. The price adjusts according to the inverse demand function p= 65 – (91 +92). Each firm has a per-unit (average) cost of 5. Each firm's payoff is its profit. a. (5 pts) Find firm l's profit as a function of qı and q2 (profit equals revenue minus total cost). b....

  • Please include the graph! Economics 302: Intermediate Microeconomics II Assignment 3 (Due in my office 3:00,...

    Please include the graph! Economics 302: Intermediate Microeconomics II Assignment 3 (Due in my office 3:00, Thursday March 19) 1. Consider two firms, 1 and 2, each producing an identical good simultaneously. This good market demand given by the (inverse demand function p = 10 - Y, where p is price, and Y = 1 + y2 is market quantity. Vi represents the amount produced by firm i. These firms have cost functions as follows: Ci = Gyi, where ci...

  • There are 2 firms in a market producing differentiated products. The firms both have MC that...

    There are 2 firms in a market producing differentiated products. The firms both have MC that is equal to 0 Firm 1 demand is q1(p1,p2) = 6-2p1 + p2 Firm 2 demand is q2(p1,p2) = 6-2p2 + p1 1. Firms compete in quantities- Cournot Competition. What are the inverse demand functions for firm 1 and 2? 2. Find and graph each firm’s best response functions. The quantities are strategic substitutes or complements? 3. Find the Nash equilibrium prices and quantities...

  • An industry consists of two Cournot firms selling a homogeneous product with a market demand curve...

    An industry consists of two Cournot firms selling a homogeneous product with a market demand curve given by P=100-Q1-Q2. Each firm has a marginal cost of $10 per unit. (a) Find the Cournot equilibrium quantities and prices. (b) What is the Bertrand equilibrium price in this market? (c) Find the quantities and price that would prevail if the firms acted as if they were a monopolist (I.e. find the collusive outcome) and then find the equilibrium price and quantity that...

  • Please show how to do this 4. Consider an economy with 2 consumption goods and N...

    Please show how to do this 4. Consider an economy with 2 consumption goods and N consumers, all with the same utility function: u(11, 12) = x 22-a, where and a € (0,1). The goods prices are pi = 2 and P2. Among the consumers, half of them each have income yi and the rest have income y2. There are m firms operating in the competitive market for good 2. Each firm has the cost function c(q) = Ba2. First,...

  • 4. Consider an economy with 2 consumption goods and N consumers, all with the same utility...

    4. Consider an economy with 2 consumption goods and N consumers, all with the same utility function: u(x1, x2) = x ma, where and a € (0,1). The goods prices are pi = 2 and P2. Among the consumers, half of them each have income yi and the rest have income y2. There are m firms operating in the competitive market for good 2. Each firm has the cost function c(q) = Bg2. First, solve for the equilibrium price P2....

  • 2. There are two firms in a market that produce an identical good, both with marginal...

    2. There are two firms in a market that produce an identical good, both with marginal cost MC=10. Fixed costs are zero for both firms. Suppose inverse demand for a product is P= 130 – e a) If the firms set the monopoly price and split the monopoly quantity. What quantities do they choose and what profit do they receive? b) Suppose they set quantities simultaneously. That is, suppose the firms play a Cournot game. What quantities do they choose...

ADVERTISEMENT
Free Homework Help App
Download From Google Play
Scan Your Homework
to Get Instant Free Answers
Need Online Homework Help?
Ask a Question
Get Answers For Free
Most questions answered within 3 hours.
ADVERTISEMENT
ADVERTISEMENT
ADVERTISEMENT